(Proof) Square of integer is 3k or 3k+1

Click For Summary
The discussion focuses on proving that the square of any integer can be expressed as either 3k or 3k+1. It starts with the division algorithm, which states that any integer can be represented as 3q, 3q+1, or 3q+2. The user attempts to calculate a^2 for each case and seeks clarification on how to derive the required forms. Responses guide the user to simplify the expressions for a^2, confirming that both cases yield results fitting the forms 3k and 3k+1. The conversation concludes with the user expressing gratitude for the clarification on the arithmetic involved.
jimmypoopins
Messages
64
Reaction score
0

Homework Statement


Prove that the square of any integer a is either of the form 3k or of the form 3k+1 for some integer k.

Homework Equations


The Division Algorithm: Let a,b be integers with b>0. Then there exists unique integers q and r such that a = bq + r and 0<=r<b

The Attempt at a Solution


I know from the division algorithm that any integer a can be written as 3q, 3q+1, or 3q+2, so
a^2=(3q)^2=9q^2=3(3q^2), or
a^2=(3q+1)^2=9q^2+6q+1, or
a^2=(3q+2)^2=9q^2+12q+4,
but i don't understand how the 3q+1 or 3q+2 case helps me.

Can anyone give me some hints or point me further in the right direction? thanks.
 
Last edited:
Physics news on Phys.org
Well it's obviously not going to be of the form 3k, right? So try to fit it to the other form, namely 3k+1. That means strip a +1 off from the +4 at the end. It should fall right out.
 
Good Work, you are basically done. Copying and pasting what you wrote,

a^2=(3q)^2=9q^2=3(3q^2), or
a^2=(3q+1)^2=9q^2+6q+1 = 3(3q^2 + 2q) + 1, or
a^2=(3q+2)^2=9q^2+12q+4 = 3(3q^2 + 4q + 1) + 1,

so you are done.
 
ircdan said:
Good Work, you are basically done. Copying and pasting what you wrote,

a^2=(3q)^2=9q^2=3(3q^2), or
a^2=(3q+1)^2=9q^2+6q+1 = 3(3q^2 + 2q) + 1, or
a^2=(3q+2)^2=9q^2+12q+4 = 3(3q^2 + 4q + 1) + 1,

so you are done.

ah, simple arithmetic :) thank you!
 
Question: A clock's minute hand has length 4 and its hour hand has length 3. What is the distance between the tips at the moment when it is increasing most rapidly?(Putnam Exam Question) Answer: Making assumption that both the hands moves at constant angular velocities, the answer is ## \sqrt{7} .## But don't you think this assumption is somewhat doubtful and wrong?

Similar threads

Replies
27
Views
3K
  • · Replies 4 ·
Replies
4
Views
3K
  • · Replies 4 ·
Replies
4
Views
2K
  • · Replies 5 ·
Replies
5
Views
3K
  • · Replies 13 ·
Replies
13
Views
4K
Replies
15
Views
4K
  • · Replies 3 ·
Replies
3
Views
956
Replies
5
Views
2K
  • · Replies 4 ·
Replies
4
Views
1K
Replies
5
Views
2K